9th Class Mathematics Triangles Some Important Results

Some Important Results

Category : 9th Class

*       Some Important Results

 

(i) The longer side of a triangle has greater angle opposite to it.

(ii) The greater angle of a triangle has longer side opposite to it.

(iii) Perpendicular line segment is the shortest in length. PM is the shortest line segment from point P to line I.

(iv) The distance between line and points lying on it is always zero.

(v) The sum of any two sides of a triangle is greater than the third side.

(vi) The difference between any two sides of a triangle is always less than its third side.    

 

 

 

 

  • Ellipses are not all similar to each other.
  • Hyperbolas are not all similar to each other.
  • In any triarigle, the three points of intersection of the adjacent angle trisectors form an equilateral triangle, called the Morley triangle.       
  • A non-planar triangle is a triangle which is not contained in a plane. 

 

 

 

  • Two triangle are said to be similar if their corresponding angles are equal and their corresponding sides are in proportion.
  • If a line is drawn parallel to any one side of a triangle intersecting the other two sides at distinct point, then it divides the two sides in the same ratio, it is known as Thale's Theorem.
  • The ratio of area of two similar triangles is equal to the ratio of the square of the corresponding sides.              
  • In a right angled triangle, the square of hypotenuse is equal to the sum of the square of the other two sides, it is known as Pythagoras Theorem.
  • Two triangles are said to be congruent if and only if their corresponding sides and angles are equal.                                   

 

 

 

 

  The quadrilateral which is formed by joining the mid points of a given quadrilateral is always_______.

(a) A parallelogram                         

(b) A trapezium

(c) A rhombus                                  

(d) A rectangle

(e) None of these  

 

Answer: (a)  

Explanation:

Given:

A quadrilateral LMNO is a quadrilateral in which P, Q, R and S are the mid points of LM, MN, NO, OL respectively.

 

To Prove:

PQRS is a parallelogram  

Construction:

Join LN  

Proof:

In\[\Delta \text{LMN}\],

\[PQ\,|\,\,|\,LN\] and \[\text{PQ}=\frac{1}{2}\text{LN}\]                         ...,.(i)

[Because P and Q are the mid points of LM and MN respectively]

In \[\Delta \text{LNO}\]

\[SR\,|\,\,|LN\] and \[\text{SR}=\frac{1}{2}\text{LN}\]             .....(ii)

[Because S and R are the mid - points of LO and ON respectively]

From (i) and (ii), we get

\[\text{PQ}=\text{SR}\] and \[~\text{PQ}\,\text{ }\!\!|\!\!\text{ }\,\,\text{ }\!\!|\!\!\text{ }\,\text{SR}\]                                           ....(iii)

Similarly we can see that

\[\text{PS}=\text{QR}\] and \[\text{PS}\,\text{ }\!\!|\!\!\text{ }\,\,\text{ }\!\!|\!\!\text{ }\,\text{QR}\]                                    .....(iv)

From (iii) and (iv), we conclude that quadrilateral PQRS is a parallelogram.  

 

 

  In \[\Delta \text{LMN}\] which LP is the bisector of MN and BR is the bisector of LP. Here, points P and R are the points of line segment MN and LN respectively then is equal to.... .

(a) \[\frac{2}{3}LN\]                                       

(b) \[\frac{1}{3}LN\]

(c) \[\frac{2}{3}LM\]                                                      

(d) \[\frac{1}{3}LM\]

(e) None of these  

 

Answer: (b)

Explanation:

Let the Q is the midpoint of LP

Draw a line \[PS\,|\,\,|\,MR\] since P is the midpoint of MN

\[\Rightarrow \]S is the midpoint of RN

In\[\Delta \text{LPN}\],

\[QR\,|\,\,|\,\,PS\]

\[\Rightarrow \] R is the midpoint of \[LS\] \[\Rightarrow \]\[LR=RS=SN\] \[\Rightarrow \]\[LR=\frac{1}{3}(LN)\]  

 

 

  In the figure given below \[\text{ON}\,\text{ }\!\!|\!\!\text{ }\,\,\text{ }\!\!|\!\!\text{ }\,\text{LM}\]

the value of\[x\]is.....

(a) 5, 11                                               

(b) 5, 8          

(c) 8, 5                                                  

(d) 18, 5

(e) None of these  

 

Answer: (d)  

 

 

  In the given \[\Delta \text{DEF},\text{ GH  }\!\!|\!\!\text{ }\,\,\text{ }\!\!|\!\!\text{  EF}\] and if \[\frac{DG}{GE}=\frac{3}{5}\] and DF = 5.6 cm then the value of DE is equal to..... .

(a) 2.1cm                                            

(b) 3.36cm      

(c) 9.35cm                                          

(d) 2.24cm

(e) None of these  

 

Answer: (a)  

 

 

  In a trapezium PQRS, PQ||RS. If the diagonal PR and QS intersect at a point 0 such that OP = 6 cm and OR = 8 cm, then the ratio of ar(\[\Delta \text{POQ}\]) and ar (\[\Delta ROS\]) is equal to:

(a) \[\frac{3}{4}\]                                            

(b) \[\frac{36}{48}\]           

(c) \[\frac{9}{16}\]                                          

(d) \[\frac{5}{16}\]

(e) None of these  

 

Answer: (c)  

 

 

  If the midpoint of the hypotenuse is joined with the vertex of a right angled triangle where the right angle formed then the length of this line segment is:

(a) \[\frac{1}{2}\] of the base                    

(b) \[\frac{1}{2}\] of the perpendicular

(c) \[\frac{1}{2}\] of the hypotenuse                     

(d) \[\frac{1}{2}\] of the area of triangle                               

(e) None of these  

 

Answer: (c)  

Explanation:  

Given:

The figure given below represents all the conditions S is the midpoint of PR,  

To prove:

 

Construction:

Produce QS to T; so that QS = ST and Join RT.  

Proof:

In\[\Delta \text{PSQ}\] and \[\Delta \text{RST}\] \[\text{PS}=\text{SR}\]                      (S is the mid point of PR)

\[\text{QS}=\text{ST}\]                               (By construction)

\[\angle \text{PSQ}=\angle \text{TSR}\]   (Vertically opposite angles)

\[\therefore \]\[\Delta \text{PSQ}=\Delta \text{RST}\]                  (By S-A-S criteria)

\[\therefore \]\[\text{PQ}=\text{RT}\]and\[\angle \text{QPR}=\angle \text{PRT}\] (Alternate interior angles)

\[\Rightarrow \]\[\text{RT}\,\text{ }\!\!|\!\!\text{ }\,\,\text{ }\!\!|\!\!\text{ }\,\text{PQ}\] \[\Rightarrow \]\[\angle \text{TRQ}=\text{9}0{}^\circ \]

Now in\[\Delta \text{PQR}\]and\[\Delta \text{TRQ}\]

\[\text{QR}=\text{QR}\]              (Common side)

\[\text{PQ}=\text{RT}\]               (Already proved)

\[\therefore \] \[\Delta PQR\cong \Delta TRS\]                 (By R-H-S criteria)

\[\Rightarrow \]\[PR=QT\]\[\Rightarrow \]\[\frac{1}{2}QT=\frac{1}{2}PR\]

 

 

  In the figure given below:

LP, MQ and NR are the median of \[\Delta \text{LMN}\] then which one of the following conditions is correct?

(a) \[(\text{LM}+\text{MN}+\text{NP})<(\text{LP}+\text{MQ}+\text{NR})\]

(b) \[\text{(LM}+\text{MN}+\text{NP})>(\text{NP}+\text{MQ}+\text{NR})\]

(c) \[\text{(LM}+\text{MN}+\text{NP})>(\text{LP}+\text{MQ}+\text{NR)}\]

(d) \[\text{(L}{{\text{M}}^{2}}+\text{M}{{\text{N}}^{2}}+\text{N}{{\text{P}}^{2}})>(\text{L}{{\text{P}}^{2}}+\text{M}{{\text{Q}}^{2}}+\text{N}{{\text{R}}^{2}}\text{)}\]

(e) None of these

 

Answer: (c)  

 

 

  One pair of vertically opposite angles of a quadrilateral are a and b respectively as shown in the figure.

Then which one of the following relations is correct?

(a) \[\text{a}+\text{b}=\text{c}+\text{d}\]                         

(b) \[\text{a}-\text{b}=\text{c}+\text{d}\]

(c) \[\text{a}+\text{b}=\text{c}-\text{d}\]                          

(d) \[\text{a}+\text{b}+\text{c}+\text{d}=\text{36}0{}^\circ \]

(e) None of these  

 

Answer: (a)  

 

 

  In the figure given below:

\[\angle \text{DEX}=\angle \text{YEX},\text{ DE}=\text{EX}\] and \[\text{EX}=\text{EF}\] then XY is equal to.....

(a) \[\angle \text{DEX}\]                             

(b) DF

(c) \[\angle \text{XEF}\]                                              

(d) EF

(e) None of these           

 

Answer: (b) 

 

 

  The line segment PQ and RS intersect each other at point 0 in such a way that OP = OS and OQ = OR then which one of the following options is correct?

(a) PR = SQ and \[~\text{PR }\!\!|\!\!\text{ }\,\,\text{ }\!\!|\!\!\text{ SQ}\]       

(b) OP=OQ and \[~\text{PR}\bcancel{\text{ }\!\!|\!\!\text{ }\,\,\text{ }\!\!|\!\!\text{ }}\text{SQ}\]

(c) RO = OS and\[~\text{PR }\!\!|\!\!\text{ }\,\,\text{ }\!\!|\!\!\text{ SQ}\]       

(d) PR = SQ and PR and SQ may or may not be parallel to each other

(e) None of these

 

Answer: (d)    

Other Topics


You need to login to perform this action.
You will be redirected in 3 sec spinner